The Official June 2015 Study Group Forum

Prepare for the LSAT or discuss it with others in this forum.
Post Reply
User avatar
RZ5646

Gold
Posts: 2391
Joined: Fri May 30, 2014 1:31 pm

Re: The Official June 2015 Study Group

Post by RZ5646 » Mon May 04, 2015 10:51 pm

appind wrote:
RZ5646 wrote:
appind wrote:anyone gets why 56.LR1.8 choice A is wrong? it appears to weaken because if "most" boulders are moved by glaciers not more than 100 miles then the boulder in stimulus is less likely to be deposited by the glacier. the same reasoning has been used in Q3 of the same section (56.LR1.Q3) to weaken, which has B as the credited answer.
But you need the AC that most weakens. AC A leaves open the possibility that it was a rare 100+ mile boulder, while D makes it impossible that it was brought by a glacier, since the glaciers move things from the north to the south.
the test instructions say most weakens to only be on the safe side. in reality, only one choice weakens on wkn qs and one choice strs on str qs.
I don't think that's true.

equang

New
Posts: 30
Joined: Tue Jan 20, 2015 12:35 am

Re: The Official June 2015 Study Group

Post by equang » Mon May 04, 2015 10:53 pm

RZ5646 wrote:
equang wrote:I'm taking the June. Did PT 56 today, still doing them untimed. Think I am finishing sections in around 35-40 mins each. Struggle with some add/fidgeting. Had the same issue on the MCAT 2 years ago
Dude if you have questions by type, use those for untimed drilling instead of actual sections. It will help you focus on one issue at a time, and you'll want real sections later for speed work.

Thank you for the advice. I do have the questions by type, though I'm not sure how to use them; I haven't been going through them wholesale. I'm not sure what issues I should focus on though. I am usually getting 92/100 raw on the untimed PTs, with 1-3 wrong per section, fluctuating. Sometimes perfect RC, sometimes perfect LG, sometimes perfect LR, other times -3.

Should I take a random sampling of my proficiency for question types by doing sets of 20-25 questions in the sorted questions? I was thinking of doing some PTs to accumulate errors and gauge my weaknesses from that. It's looking like Weaken questions are a soft spot, other errors I make tend to be brain farts.

Should I just drill for the sake of drilling? I am averaging 170-175 on untimed PTs from 45 to 55, and am looking to get to 175-180.

Any help will be appreciated!


To the other posters : Don't doubt yourselves. The scores you want are just a few questions away.

User avatar
appind

Gold
Posts: 2266
Joined: Mon Nov 12, 2012 3:07 am

Re: The Official June 2015 Study Group

Post by appind » Mon May 04, 2015 10:53 pm

RZ5646 wrote:
appind wrote:
RZ5646 wrote:
appind wrote:anyone gets why 56.LR1.8 choice A is wrong? it appears to weaken because if "most" boulders are moved by glaciers not more than 100 miles then the boulder in stimulus is less likely to be deposited by the glacier. the same reasoning has been used in Q3 of the same section (56.LR1.Q3) to weaken, which has B as the credited answer.
But you need the AC that most weakens. AC A leaves open the possibility that it was a rare 100+ mile boulder, while D makes it impossible that it was brought by a glacier, since the glaciers move things from the north to the south.
the test instructions say most weakens to only be on the safe side. in reality, only one choice weakens on wkn qs and one choice strs on str qs.
I don't think that's true.
http://www.top-law-schools.com/forums/v ... +#p7894153

http://www.top-law-schools.com/forums/v ... +#p3791546

also BP thread imo
bp thread wrote: I think people have trouble with strengthen/weaken questions in part because they're unsure about the task. It might seem like there should be some amount of grey area to compare answer choices that way--like, does A strengthen the argument more than B? But it will absolutely never happen that you're comparing two strengtheners or weakeners, and the correct answer comes down to a matter of degree. Despite the fact that the stem asks which choice "most" strengthens/weakens the argument, that is just not how the LSAT works. Exactly one answer choice will strengthen/weaken the argument (either a lot or a little), and exactly four will NOT strengthen/weaken it at all. So your main job is really to figure out why four of the answer choices have no effect, or the opposite effect, on the argument in relation to the task.
Last edited by appind on Mon May 04, 2015 10:57 pm, edited 1 time in total.

User avatar
RZ5646

Gold
Posts: 2391
Joined: Fri May 30, 2014 1:31 pm

Re: The Official June 2015 Study Group

Post by RZ5646 » Mon May 04, 2015 10:56 pm

Strengthen, weaken, and parallel are usually the hardest for LR. I don't really drill LR so I don't have a ton of advice there. It sounds like you probably have the basics down, so do some timed sections (or a timed PT) and see how you do; the results will help guide your drilling.

User avatar
JackelJ

Silver
Posts: 1404
Joined: Sun Aug 10, 2014 6:47 pm

Re: The Official June 2015 Study Group

Post by JackelJ » Mon May 04, 2015 11:02 pm

Re strengthen/weaken debate: as is true for all other types of questions, there is only one answer choice that strengthens or weakens and the other 4 do not. "Most strengthens" doesn't mean there's one answer that's better than another answer that can sort of strengthen, it means find the only AC that strengthens. I don't have the specific question in front of me to look at but you can try Manhattan forums or Velocity videos, or maybe LSAT Hacks to see their explanations.

Want to continue reading?

Register now to search topics and post comments!

Absolutely FREE!


User avatar
RZ5646

Gold
Posts: 2391
Joined: Fri May 30, 2014 1:31 pm

Re: The Official June 2015 Study Group

Post by RZ5646 » Mon May 04, 2015 11:11 pm

appind wrote:
http://www.top-law-schools.com/forums/v ... +#p7894153

http://www.top-law-schools.com/forums/v ... +#p3791546

also BP thread imo
bp thread wrote: I think people have trouble with strengthen/weaken questions in part because they're unsure about the task. It might seem like there should be some amount of grey area to compare answer choices that way--like, does A strengthen the argument more than B? But it will absolutely never happen that you're comparing two strengtheners or weakeners, and the correct answer comes down to a matter of degree. Despite the fact that the stem asks which choice "most" strengthens/weakens the argument, that is just not how the LSAT works. Exactly one answer choice will strengthen/weaken the argument (either a lot or a little), and exactly four will NOT strengthen/weaken it at all. So your main job is really to figure out why four of the answer choices have no effect, or the opposite effect, on the argument in relation to the task.
I still disagree, especially since the argument in question is explicitly probabilistic, but it doesn't really matter because there's always one AC that's a much better strengthener/weakener than any other, signaling that it's TCR.

User avatar
JaimeD15

Bronze
Posts: 156
Joined: Tue Mar 10, 2015 7:11 pm

Re: The Official June 2015 Study Group

Post by JaimeD15 » Mon May 04, 2015 11:18 pm

Finished the trainer on Saturday. Working on a PT now, kind of scared at the results but I'm right on track with the 12 week curriculum from the trainer/integrating the LGB. Drilling and PTs are all my life consists of for the next 35 days.

How is everyone else holding up?

User avatar
appind

Gold
Posts: 2266
Joined: Mon Nov 12, 2012 3:07 am

Re: The Official June 2015 Study Group

Post by appind » Mon May 04, 2015 11:19 pm

RZ5646 wrote:
appind wrote:
http://www.top-law-schools.com/forums/v ... +#p7894153

http://www.top-law-schools.com/forums/v ... +#p3791546

also BP thread imo
bp thread wrote: I think people have trouble with strengthen/weaken questions in part because they're unsure about the task. It might seem like there should be some amount of grey area to compare answer choices that way--like, does A strengthen the argument more than B? But it will absolutely never happen that you're comparing two strengtheners or weakeners, and the correct answer comes down to a matter of degree. Despite the fact that the stem asks which choice "most" strengthens/weakens the argument, that is just not how the LSAT works. Exactly one answer choice will strengthen/weaken the argument (either a lot or a little), and exactly four will NOT strengthen/weaken it at all. So your main job is really to figure out why four of the answer choices have no effect, or the opposite effect, on the argument in relation to the task.
I still disagree, especially since the argument in question is explicitly probabilistic, but it doesn't really matter because there's always one AC that's a much better strengthener/weakener than any other, signaling that it's TCR.
hmm, so what's your position. is it that the argument is probabilistic and therefore A doesn't weaken at all, or is it that A weakens but less so than the TCR? it is widely acknowledged amongst lsat gurus that only one choice wks or strs in wkn/str qs respectively, that's just the way the test is designed.

User avatar
Clyde Frog

Platinum
Posts: 8985
Joined: Sun May 26, 2013 2:27 am

Re: The Official June 2015 Study Group

Post by Clyde Frog » Mon May 04, 2015 11:22 pm

equang wrote:
RZ5646 wrote:
equang wrote:I'm taking the June. Did PT 56 today, still doing them untimed. Think I am finishing sections in around 35-40 mins each. Struggle with some add/fidgeting. Had the same issue on the MCAT 2 years ago
Dude if you have questions by type, use those for untimed drilling instead of actual sections. It will help you focus on one issue at a time, and you'll want real sections later for speed work.

Thank you for the advice. I do have the questions by type, though I'm not sure how to use them; I haven't been going through them wholesale. I'm not sure what issues I should focus on though. I am usually getting 92/100 raw on the untimed PTs, with 1-3 wrong per section, fluctuating. Sometimes perfect RC, sometimes perfect LG, sometimes perfect LR, other times -3.

Should I take a random sampling of my proficiency for question types by doing sets of 20-25 questions in the sorted questions? I was thinking of doing some PTs to accumulate errors and gauge my weaknesses from that. It's looking like Weaken questions are a soft spot, other errors I make tend to be brain farts.

Should I just drill for the sake of drilling? I am averaging 170-175 on untimed PTs from 45 to 55, and am looking to get to 175-180.

Any help will be appreciated!


To the other posters : Don't doubt yourselves. The scores you want are just a few questions away.

Take the tests timed and then do them untimed afterwards.

Want to continue reading?

Register for access!

Did I mention it was FREE ?


User avatar
whacka

Gold
Posts: 1634
Joined: Tue Apr 21, 2015 11:46 pm

Re: The Official June 2015 Study Group

Post by whacka » Mon May 04, 2015 11:24 pm

.
Last edited by whacka on Sat Jul 11, 2015 2:30 pm, edited 1 time in total.

User avatar
RZ5646

Gold
Posts: 2391
Joined: Fri May 30, 2014 1:31 pm

Re: The Official June 2015 Study Group

Post by RZ5646 » Mon May 04, 2015 11:30 pm

Our difference seems to stem from the meaning of "weaken" and whether making something seem unlikely is a form of weakening. It seems odd to me that LSAC would use the "most" phrasing if they didn't share my view, but like I said, this is all academic because one can answer these questions correctly regardless of one's view on this matter.

User avatar
Christine (MLSAT)

Bronze
Posts: 357
Joined: Fri Nov 22, 2013 3:41 pm

Re: The Official June 2015 Study Group

Post by Christine (MLSAT) » Mon May 04, 2015 11:31 pm

appind wrote:anyone gets why 56.LR1.8 choice A is wrong? it appears to weaken because if "most" boulders are moved by glaciers not more than 100 miles then the boulder in stimulus is less likely to be deposited by the glacier. the same reasoning has been used in Q3 of the same section (56.LR1.Q3) to weaken, which has B as the credited answer.

Classic trap my friend.

Saying that most glacier-moved-boulders are moved <100miles doesn't make it less likely that this >100mile boulder is glacier-moved.

Take a silly example. Let's say you get a 180 on the LSAT. You'd like to know your chances of getting into Yale. Do you care what proportion of the Yale accepteds have 180s? Or should you care what proportion of 180s that applied were accepted to Yale? Maybe there are only 3 180s accepted to Yale this year - a tiny fraction of the accepteds. But if only 3 180s applied, that's a 100% acceptance rate for 180s.

What if there are 100 glacier-moved-boulders in the world, and only 10 of them were moved >100miles. But what if those 10 boulders are the only >100mile boulders that have ever existed? Out of the >100mile boulders, all of them were glacier moved, making it highly likely that *this* >100mile is also glacier moved. It doesn't matter that most of the glacier-moved-boulders do something else.

I don't care what percentage of glacier-moved-boulders is >100miles. I'd care what percentage of >100miles boulders are glacier moved.

This kind of thing shows up a ton on inference questions, where 'most' statements are a centerpiece of the stimulus.

If you want to shorthand it:

Most A are B.
Bob is A.
Valid inference: it's likely that Bob is B.


Most A are B.
Bob is B.
INVALID inference: it's likely that Bob is A.


This answer does a variation on the above with:

Most A are not B
Boulder is B.
Tempting-but-invalid inference: it's likely boulder is not A.

User avatar
appind

Gold
Posts: 2266
Joined: Mon Nov 12, 2012 3:07 am

Re: The Official June 2015 Study Group

Post by appind » Mon May 04, 2015 11:31 pm

whacka wrote:
appind wrote:
RZ5646 wrote:
I still disagree, especially since the argument in question is explicitly probabilistic, but it doesn't really matter because there's always one AC that's a much better strengthener/weakener than any other, signaling that it's TCR.
hmm, so what's your position. is it that the argument is probabilistic and therefore A doesn't weaken at all, or is it that A weakens but less so than the TCR? it is widely acknowledged amongst lsat gurus that only one choice wks or strs in wkn/str qs respectively, that's just the way the test is designed.
yeah they don't design the test to have you weigh which one is better, it's definitely 4 wrong answers and 1 right ones. The reason that A is wrong is that saying "well that's a pretty rare occurrence" doesn't undermine a conclusion at all. If it was "glaciers cannot push rocks that far" then it would undermine. Saying "glaciers usually don't" doesn't do anything.
why then does B weaken in Q3 of the same section? that seems to wkn using the same reasoning as 8.A by applying "regularly" in the case of Ms Garon.

ps: agree on 4 non-wkners and 1 wknr.

Register now!

Resources to assist law school applicants, students & graduates.

It's still FREE!


User avatar
RZ5646

Gold
Posts: 2391
Joined: Fri May 30, 2014 1:31 pm

Re: The Official June 2015 Study Group

Post by RZ5646 » Mon May 04, 2015 11:36 pm

appind wrote:
whacka wrote:
appind wrote:
RZ5646 wrote:
I still disagree, especially since the argument in question is explicitly probabilistic, but it doesn't really matter because there's always one AC that's a much better strengthener/weakener than any other, signaling that it's TCR.
hmm, so what's your position. is it that the argument is probabilistic and therefore A doesn't weaken at all, or is it that A weakens but less so than the TCR? it is widely acknowledged amongst lsat gurus that only one choice wks or strs in wkn/str qs respectively, that's just the way the test is designed.
yeah they don't design the test to have you weigh which one is better, it's definitely 4 wrong answers and 1 right ones. The reason that A is wrong is that saying "well that's a pretty rare occurrence" doesn't undermine a conclusion at all. If it was "glaciers cannot push rocks that far" then it would undermine. Saying "glaciers usually don't" doesn't do anything.
why then does B weaken in Q3 of the same section? that seems to wkn using the same reasoning as 8.A by applying "regularly" in the case of Ms Garon.

ps: agree on 4 non-wkners and 1 wknr.
That's what I was going to say: you can probably find hard weaken questions where TCR is just as feeble at weakening as A is in question 8 but is TCR nonetheless because out of the available choices it "most weakens."

B in question 3 is exactly such a TCR that would be brushed aside as not really weakening if there were a better option available.

User avatar
whacka

Gold
Posts: 1634
Joined: Tue Apr 21, 2015 11:46 pm

Re: The Official June 2015 Study Group

Post by whacka » Mon May 04, 2015 11:41 pm

.
Last edited by whacka on Sat Jul 11, 2015 2:29 pm, edited 1 time in total.

User avatar
RZ5646

Gold
Posts: 2391
Joined: Fri May 30, 2014 1:31 pm

Re: The Official June 2015 Study Group

Post by RZ5646 » Mon May 04, 2015 11:43 pm

Well the only thing that matters is whether you get the questions right, and my LR average is like -0.5, so I think I understand it well enough.

And LSAC wouldn't need to cover their asses if those hypothetical complaints didn't have some validity, so I think that just backs me up.
Last edited by RZ5646 on Mon May 04, 2015 11:45 pm, edited 1 time in total.

User avatar
Christine (MLSAT)

Bronze
Posts: 357
Joined: Fri Nov 22, 2013 3:41 pm

Re: The Official June 2015 Study Group

Post by Christine (MLSAT) » Mon May 04, 2015 11:45 pm

appind wrote: why then does B weaken in Q3 of the same section? that seems to wkn using the same reasoning as 8.A by applying "regularly" in the case of Ms Garon.

ps: agree on 4 non-wkners and 1 wknr.
RZ5646 wrote: That's what I was going to say: you can probably find hard weaken questions where TCR is just as feeble at weakening as A is in question 8 but is TCR nonetheless because out of the available choices it "most weakens."

B in question 3 is exactly such a TCR that would be brushed aside as not really weakening if there were a better option available.

Sorry RZ, I have to disagree with you here. The difference is not that TCR for Q3 sucks, but you have no better options. You have a totally different job in Q3.

Take a look at the assumption that's being made in Q3 - for this argument to even remotely make sense, the author is assuming that Ms. Garon must have been an unskilled worker. If she wasn't, using her as an example is ridiculous, and the argument would fall apart. That assumption is very, very strong - as a result, it's very easy to attack. All I need is to raise the possibility that Ms. Garon might have been skilled, and I've damaged that super-strong assumption.

The boulders question is totally different; the conclusion is a statement of likelihood. Here, an answer that simply said that 'some boulders' were a certain way would do nothing. "Some" statements can't really help or hurt a softer conclusion that is just saying X is more likely than Y. A 'most' statement can absolutely help, though - it just has to be the *right* 'most' statement, and that's where (A) fails.

Strength of language does matter in strengthen/weaken questions, but it's not a universal indication - it depends on the strength of the assumption/conclusion in play.
Last edited by Christine (MLSAT) on Mon May 04, 2015 11:49 pm, edited 1 time in total.

Get unlimited access to all forums and topics

Register now!

I'm pretty sure I told you it's FREE...


User avatar
whacka

Gold
Posts: 1634
Joined: Tue Apr 21, 2015 11:46 pm

Re: The Official June 2015 Study Group

Post by whacka » Mon May 04, 2015 11:47 pm

.
Last edited by whacka on Sat Jul 11, 2015 2:29 pm, edited 2 times in total.

User avatar
JackelJ

Silver
Posts: 1404
Joined: Sun Aug 10, 2014 6:47 pm

Re: The Official June 2015 Study Group

Post by JackelJ » Mon May 04, 2015 11:53 pm

Excellent responses Christine. I started to type out responses but you had much quicker and better quality responses than mine would have been.

User avatar
RZ5646

Gold
Posts: 2391
Joined: Fri May 30, 2014 1:31 pm

Re: The Official June 2015 Study Group

Post by RZ5646 » Mon May 04, 2015 11:54 pm

Yeah I see what you mean: 3 is about reasoning, while 8 is about the likelihood of the conclusion given a set of facts. That is a relevant difference.

User avatar
appind

Gold
Posts: 2266
Joined: Mon Nov 12, 2012 3:07 am

Re: The Official June 2015 Study Group

Post by appind » Tue May 05, 2015 12:00 am

Christine (MLSAT) wrote:
appind wrote: why then does B weaken in Q3 of the same section? that seems to wkn using the same reasoning as 8.A by applying "regularly" in the case of Ms Garon.

ps: agree on 4 non-wkners and 1 wknr.
RZ5646 wrote: That's what I was going to say: you can probably find hard weaken questions where TCR is just as feeble at weakening as A is in question 8 but is TCR nonetheless because out of the available choices it "most weakens."

B in question 3 is exactly such a TCR that would be brushed aside as not really weakening if there were a better option available.

Sorry RZ, I have to disagree with you here. The difference is not that TCR for Q3 sucks, but you have no better options. You have a totally different job in Q3.

Take a look at the assumption that's being made in Q3 - for this argument to even remotely make sense, the author is assuming that Ms. Garon must have been an unskilled worker. If she wasn't, using her as an example is ridiculous, and the argument would fall apart. That assumption is very, very strong - as a result, it's very easy to attack. All I need is to raise the possibility that Ms. Garon might have been skilled, and I've damaged that super-strong assumption.

The boulders question is totally different; the conclusion is a statement of likelihood. Here, an answer that simply said that 'some boulders' were a certain way would do nothing. "Some" statements can't really help or hurt a softer conclusion that is just saying X is more likely than Y. A 'most' statement can absolutely help, though - it just has to be the *right* 'most' statement, and that's where (A) fails.

Strength of language does matter in strengthen/weaken questions, but it's not a universal indication - it depends on the strength of the assumption/conclusion in play.
if Q8 conclusion wasn't a statement of likelihood, then there would be no difference between the reasoning in q8.A and q3.B.
modifying q8 as below, "boulder is likely not A" is still an invalid inference and therefore wouldn't weaken. yet the modified reasoning is the same as q3.B. how does that reconcile?

q8:
choice-A: Most A are not B
conclusion: Boulder is likely B.
Tempting-but-invalid inference: boulder is likely not A.

q8 modified:
choice-A: Most A are not B
modified conclusion: Boulder is B
Tempting-but-invalid inference: boulder is likely not A.
Last edited by appind on Tue May 05, 2015 12:22 am, edited 1 time in total.

Communicate now with those who not only know what a legal education is, but can offer you worthy advice and commentary as you complete the three most educational, yet challenging years of your law related post graduate life.

Register now, it's still FREE!


User avatar
Christine (MLSAT)

Bronze
Posts: 357
Joined: Fri Nov 22, 2013 3:41 pm

Re: The Official June 2015 Study Group

Post by Christine (MLSAT) » Tue May 05, 2015 12:21 am

Appind, [edit] I think I see your issue now. You are thinking the difference between the two questions is just one of strength of language. I can see why you thought that! But there's a bigger difference in play. [/edit]

The post of mine that you quoted was meant to show that we CAN'T dismiss (A) on 8 out of hand just because it's a 'most' statement. The strength of 'most' is absolutely enough to matter, if it's the right kind of 'most' statement.

The problem with (A) is NOT strength of language - it's that it's the wrong kind of 'most' statement. That's what I was trying to explain in my first post. The same issues would apply if we shifted the strengths to match Q3.

If we changed the conclusion to say "this boulder was definitely deposited by a glacier", then a valid weakener would be "some boulders get moved 100s of miles by things other than glaciers." What wouldn't be a valid weakener would be "Some boulders moved by glaciers are moved less than 100 miles."

To relate it back to Q3, the valid weakener (B) can be paraphrase "some skilled workers get hired in entry level positions." What would not have been a valid weakener would have been "some unskilled workers are rejected for entry level positions."

User avatar
appind

Gold
Posts: 2266
Joined: Mon Nov 12, 2012 3:07 am

Re: The Official June 2015 Study Group

Post by appind » Tue May 05, 2015 12:42 am

Christine (MLSAT) wrote:Appind, [edit] I think I see your issue now. You are thinking the difference between the two questions is just one of strength of language. I can see why you thought that! But there's a bigger difference in play. [/edit]

The post of mine that you quoted was meant to show that we CAN'T dismiss (A) on 8 out of hand just because it's a 'most' statement. The strength of 'most' is absolutely enough to matter, if it's the right kind of 'most' statement.

The problem with (A) is NOT strength of language - it's that it's the wrong kind of 'most' statement. That's what I was trying to explain in my first post. The same issues would apply if we shifted the strengths to match Q3.

If we changed the conclusion to say "this boulder was definitely deposited by a glacier", then a valid weakener would be "some boulders get moved 100s of miles by things other than glaciers." What wouldn't be a valid weakener would be "Some boulders moved by glaciers are moved less than 100 miles."

To relate it back to Q3, the valid weakener (B) can be paraphrase "some skilled workers get hired in entry level positions." What would not have been a valid weakener would have been "some unskilled workers are rejected for entry level positions."
i am only using the example of invalid inference in your post reproduced below to show that in q8 using A to weaken the conclusion would be an invalid inference. but if one modified the conclusion of q8 to be definitive and not probabilistic, then it'd look like the following.

q8 modified:
choice-A: Most A are not B
modified conclusion: Boulder is B
Tempting-but-invalid inference: boulder is likely not A.

[e:clarity]
q8 modified:
choice-A: Most A are not B (B= >100miles; A=glacier moved)
premise: Boulder is B
modified conclusion: Boulder is A
Tempting-but-invalid inference: boulder is likely not A.
[/e]


in this case, it'd still be wrong to think that A weakened this modified conclusion as that'd be an invalid inference.

q3:
assumption: Garon is B (B means unskilled)
choice B: most A are not B

invalid inference: Garon is likely A

B is weakening the assumption through this invalid inference here. contradiction?
Christine (MLSAT) wrote:
appind wrote:anyone gets why 56.LR1.8 choice A is wrong? it appears to weaken because if "most" boulders are moved by glaciers not more than 100 miles then the boulder in stimulus is less likely to be deposited by the glacier. the same reasoning has been used in Q3 of the same section (56.LR1.Q3) to weaken, which has B as the credited answer.

Classic trap my friend.

Saying that most glacier-moved-boulders are moved <100miles doesn't make it less likely that this >100mile boulder is glacier-moved.

Take a silly example. Let's say you get a 180 on the LSAT. You'd like to know your chances of getting into Yale. Do you care what proportion of the Yale accepteds have 180s? Or should you care what proportion of 180s that applied were accepted to Yale? Maybe there are only 3 180s accepted to Yale this year - a tiny fraction of the accepteds. But if only 3 180s applied, that's a 100% acceptance rate for 180s.

What if there are 100 glacier-moved-boulders in the world, and only 10 of them were moved >100miles. But what if those 10 boulders are the only >100mile boulders that have ever existed? Out of the >100mile boulders, all of them were glacier moved, making it highly likely that *this* >100mile is also glacier moved. It doesn't matter that most of the glacier-moved-boulders do something else.

I don't care what percentage of glacier-moved-boulders is >100miles. I'd care what percentage of >100miles boulders are glacier moved.

This kind of thing shows up a ton on inference questions, where 'most' statements are a centerpiece of the stimulus.

If you want to shorthand it:

Most A are B.
Bob is A.
Valid inference: it's likely that Bob is B.


Most A are B.
Bob is B.
INVALID inference: it's likely that Bob is A.


This answer does a variation on the above with:

Most A are not B
Boulder is B.
Tempting-but-invalid inference: it's likely boulder is not A.
Last edited by appind on Tue May 05, 2015 12:19 pm, edited 1 time in total.

User avatar
Christine (MLSAT)

Bronze
Posts: 357
Joined: Fri Nov 22, 2013 3:41 pm

Re: The Official June 2015 Study Group

Post by Christine (MLSAT) » Tue May 05, 2015 1:12 am

Okay, I think I see where you are getting turned around.

First, you're 100% right that Q8.(A) would not weaken the modified conclusion either. There's no valid inference I can shake out from (A) that would weaken either a probabilistic or a definitive conclusion that the boulder (probably) is glacier-moved.

You're also 100% correct that on Q3 it would be invalid to conclude that Garon is LIKELY skilled. Fortunately, though, we don't need to go that far to find an inference that would weaken the argument.

It is totally valid to conclude from Q3.(B) that some entry-level positions are staffed by skilled workers. That very small valid inference is enough to damage the assumption that Garon absolutely must have been unskilled.

If I assumed that all cats were orange, even one instance of a non-orange cat would be enough to weaken my argument.

Premise: Fluffy is a cat.
Conclusion: Therefore, Fluffy is orange.
Assumption: All cats are orange.
Valid weakener: There are some cats in the world that aren't orange.




(As a side note, I don't trust the word 'regularly' to mean 'most'. It surely means at least 'some', but I wouldn't place bets past that.)

User avatar
biggestlawman

Silver
Posts: 650
Joined: Mon Mar 02, 2015 4:29 pm

Re: The Official June 2015 Study Group

Post by biggestlawman » Tue May 05, 2015 11:24 am

So seems the new gig will begin coming Monday. Life will get very busy with work and LSAT. I am planning on 3 hours of studying every weekday and 5 hours every weekend day from next week with a 3 week vacation in a month's time during which I will study 2 hours a day. I have started posting in the October thread!

The vacation is in France, Greece and Turkey! :D

Seriously? What are you waiting for?

Now there's a charge.
Just kidding ... it's still FREE!


Post Reply

Return to “LSAT Prep and Discussion Forum”